Questions tagged [distribution-theory]

Use this tag for questions about distributions (or generalized functions). For questions about "probability distributions", use (probability-distributions). For questions about distributions as sub-bundles of a vector bundle, use (differential-geometry).

Filter by
Sorted by
Tagged with
2 votes
1 answer
360 views

What would be to show $(1+|\xi|^2)^{s/2}\hat{u}\in \mathcal{S}^{'}(\mathbb R^n)$ is a function?

If $s\in\mathbb R$ define the sobolev space $H^s(\mathbb R^n)$ as $$H^s(\mathbb R^n):=\{u\in \mathcal{S}^{'}(\mathbb R^n): (1+|\xi|^2)^{s/2}\hat{u}\in L^2(\mathbb R^n)\}.$$ I have a doubt concearning ...
PtF's user avatar
  • 9,655
2 votes
1 answer
1k views

Distribution with finite support. [duplicate]

If $f \in\mathcal D′(\Bbb R^n)$, is a distribution and support of this distribution is a set of finite points in $\Bbb R^n$. Can anyone tell me that what will be the general form of this ...
Sara Tancredi's user avatar
2 votes
2 answers
76 views

Convergence of a sequence in $L^1(\mathbb{R}^3)$

All function spaces are over $\mathbb{R}^3$. Let $u_n \in C^\infty_0$, $u_n\rightarrow u$ in $L^1$. Let $v\in L^1_\text{loc}$ be such that $uv \in L^1$. Does $u_n v \rightarrow uv$ in $L^1$? What ...
Jas Ter's user avatar
  • 1,539
2 votes
1 answer
3k views

Proving the mean value property of harmonic functions using distributions?

A professor I talked to showed me a proof of the mean value property. (He actually showed it for functions solving the heat equation instead of Laplace's equation, but it seems like the argument is ...
Alan C's user avatar
  • 2,020
2 votes
1 answer
557 views

What does it mean to say an integral exists 'in the distributional sense'?

What exactly does it mean to say that an integral exists 'just in the distributional sense'? For example, the Fourier transform of $x^2 e^{-\lambda x}$ or of $H(R-|x|)$ where $R > 0$ and $H$ is the ...
StefanH's user avatar
  • 18.1k
2 votes
1 answer
603 views

Proving that the bi-laplacian of a radial basis function is the dirac delta

According to equation (2.14) of the paper "The Uniform Convergence of Thin Plate Spline Interpolation in Two Dimensions" a radial basis function $\phi(\parallel x \parallel)$ has the property $$ \...
Olumide's user avatar
  • 1,241
2 votes
1 answer
76 views

Differential equation with Fourier transform

It is given the following differential equation: $$-\frac{d^2 x}{dt^2} + \frac{dx}{dt} = \theta(t) e^{-\varepsilon t}$$ where $\theta(t)$ is the Heaviside function (which equals $0$ for $t<0$ and ...
deomanu01's user avatar
2 votes
1 answer
71 views

What Hardy's inequality is this?

During a seminar the other day, the lecturer mentioned a Hardy's inequality $|x|^{-2} \le K(1-\Delta)$, where $K$ is a constant and $\Delta$ is the Laplacian in $\mathbb{R}^{3}$. I searched for Hardy'...
InMathweTrust's user avatar
2 votes
1 answer
111 views

Why is the solution of the Klein-Gordon PDE a distribution?

I've also posted this question on physics SE in case it is more appropiate there. Consider the Klein-Gordon equation: $$(\square + m^2)\phi = (\partial_t^2 - \Delta + m^2)\phi = 0 \tag{1}$$ The ...
CBBAM's user avatar
  • 5,965
2 votes
1 answer
88 views

Are all distributions of the form $ \varphi \mapsto \int \varphi^{(n)} d\mu $?

Distributions are defined as the set of linear continuos operators over the set of test functions $ \mathcal{D}(\mathbb{R}) = C^{\infty}_c(\mathbb{R}) $, i.e. $ \mathcal{D}(\mathbb{R})^* $. My ...
rod's user avatar
  • 769
2 votes
1 answer
35 views

derivative of the functions in Hadamard's lemma

I think Corollary 1.10 of this PDF page 12 is Hadamard's lemma here $\partial_jf$ is the partial derivative $\frac{\partial f}{\partial x_j}$ and $\alpha$ is a multi-index. $\partial^\alpha$ is ...
hbghlyj's user avatar
  • 2,345
2 votes
1 answer
62 views

The derivative of the Heaviside step function in $\mathbb{R}^n$

Here is a problem I have: Consider the Heaviside function $$H(x)=H(x_1)H(x_2)\cdots H(x_n), ~~~x\in\mathbb{R}^n$$ and prove that $$\dfrac{\partial^n H(x)}{\partial x_1\partial x_2\cdots\partial x_n}=\...
Berban's user avatar
  • 29
2 votes
1 answer
41 views

Distribution of the combination of square and the product of Gaussian random variables [closed]

What is the distribution of the following expression? $$ \sum_{i=1}^n a_i x_i^2 + \sum_{i \neq j; i,j=1}^n b_{ij} x_i x_j $$ where $x_i$ for $i$ in the range from 1 to $n$ are i.i.d. samples from $\...
Bhisham's user avatar
  • 219
2 votes
1 answer
55 views

Distribution against non-zero function

I often see equation following: $$\int_{-\infty}^{\infty} \delta(x) dx=1 =\int_{-\infty}^{\infty} 1 \cdot \delta(x) dx $$ But, as i know, The distribution isn't possible for any function; it is only ...
KHJ's user avatar
  • 69
2 votes
1 answer
98 views

Translation operator on tempered distributions converges to derivative operator

Let $\mathscr{S}(\mathbb{R})$ be the Schwartz space over $\mathbb{R}$ and $\mathscr{S}'(\mathbb{R})$ the space of tempered distributions. Define an operator $U_a$ on the space of tempered ...
CBBAM's user avatar
  • 5,965
2 votes
1 answer
48 views

What is the Laplace transform of the distribution $\delta \left(\cos \left(\frac{1}{x}\right)\right)$?

Trying to find numerocity of the roots of the function $\cos(1/x)$ I stumbled at the following problem: what is the Laplace transform of the distribution $\delta \left(\cos \left(\frac{1}{x}\right)\...
Anixx's user avatar
  • 9,119
2 votes
1 answer
594 views

Integrating the dirac delta function multiplied with another function

I'm trying to evaluate $$ \int_{-\infty}^{\infty}dxf(x)\delta(g(x)) $$ Where $\delta$ is the dirac delta function, and $g(x)$ has zeros at $\{x=x_n, n = 1,...,N\}$. Here're some of my steps: $$ \int_{-...
IGY's user avatar
  • 919
2 votes
2 answers
57 views

$ \frac{1}{2\pi } \int dk e^{ik x } = \delta(x)$ in the distributional sense

It is often said or heard. The right hand side is well understood as a distribution or linear functional. How to make sense of the left hand side as a distribution?
poisson's user avatar
  • 1,005
2 votes
1 answer
51 views

Prove that $\eta^\epsilon(x)$ converges to Dirac delta

I would like to prove that $$\eta^\epsilon(x):=\begin{cases}\displaystyle\frac{\epsilon-|x|}{\epsilon^2}&\text{, if }|x|\le \epsilon\\0&\text{, if }|x|\ge \epsilon\end{cases}$$ converges to ...
Mark's user avatar
  • 7,851
2 votes
1 answer
330 views

Fourier transformation of tempered distribution $p.v. \frac{1}{x}$

Prove the fourier transformation of $p.v. \frac{1}{x}\in \mathcal{S}'(\Bbb{R})$ is $-i\pi \text{sgn}(y)$. This is a classical result that shows in many textbook,my question is some detail question ...
yi li's user avatar
  • 4,786
2 votes
2 answers
192 views

What is the "Main Value of an Integral" (denoted as $\textbf{VP}\int_{-\infty}^{\infty} \frac{\phi(x)}{x}dx$ for $f(x) = \frac{\phi(x)}{x}$)?

The definition given to us is as follows (for $\text{supp }\phi \subset (-R, R)$ ): $$\textbf{VP }\int_{-\infty}^{\infty} \frac{\phi(x)}{x}dx = \lim_{\varepsilon \rightarrow 0^+} \: (\int_{-R}^{-\...
anon's user avatar
  • 555
2 votes
1 answer
35 views

$C^{\infty}$ reconciliation of two functions

Let $\Omega$ be an open subset of $\mathbb{R}^d$, $K$ a compact subset of $\Omega$ and $\phi \in C^\infty(\Omega)$. Given $\epsilon > 0$ and denoting $K_\epsilon = \{ x \in K : d(x,K^c) \geq \...
blamethelag's user avatar
  • 2,007
2 votes
1 answer
101 views

Weak convergence and Laplace's equation

Suppose $\{u_i\}$ is a sequence of distributions that satisfies Laplace's equation $$\Delta u_i = 0, \forall i$$ in the weak sense. If $\{u_i\} \to u$ in the sense of distribution, show that $u$ also ...
Morcus's user avatar
  • 583
2 votes
1 answer
396 views

What are distributions really?

If they are considered to be generalisation of function, as for examples real numbers are for rationals, then it must be possible too reinterpret any “classical” function $f(x)$ in terms of a ...
ric.san's user avatar
  • 121
2 votes
2 answers
147 views

Show that the finite part of $\frac{1}{x_+}$ is a distribution

I would like to show that the finite part of $\frac{1}{x_+}=\mathbb{1}_{[0,+\infty[}\frac{1}{x}$, noted Fp$\left(\frac{1}{x_+}\right)$ and given by $$\left\langle\text{Fp}\left(\frac{1}{x_+}\right),\...
QGM's user avatar
  • 613
2 votes
1 answer
81 views

If $u \in C$, $|\int u \phi_{ij}| \leq 2 \|h\|_p \|\phi\|_{p'} \forall \phi \in C_c^\infty$, then $\|u_{ij}\|_p \leq 2\|h\|_p$?

Let $\Omega$ be a domain in $\mathbb R^N$, $u \in C(\Omega)$ and $h \in L^p(\Omega)$. Suppose that $$ \left|\int_\Omega u \varphi_{ij} \ dx \right| \leq 2\|h\|_p \|\varphi\|_{p'} \quad \forall \...
Danilo Gregorin Afonso's user avatar
2 votes
2 answers
544 views

Dirac Delta Function "removing integral"

I am trying to proof the following identity: \begin{align} \delta(ax) = \frac{1}{|a|}\delta(x). \end{align} where $\delta$ represents the dirac delta function. I found that \begin{align} \int_{-\...
Philxy's user avatar
  • 21
2 votes
2 answers
137 views

About the definition of $\phi_k\xrightarrow{D}\phi$ (distribution theory)

Let $\phi\in \mathcal{D}(\Omega):=C_c^{\infty}(\Omega)$ and $(\phi_k)_k$ a sequence of functions in $\mathcal{D}(\Omega)$ We define $\varphi_k \rightarrow \varphi$ in $\mathcal{D}(\Omega)$ as (1) $\...
roi_saumon's user avatar
  • 4,206
2 votes
1 answer
126 views

Proving a sequence of locally integrable functions goes to Dirac delta

Attempt: I have done this type of questions before too and usually a substitution $x =\frac{x}{m}$ does the job. But it does not work in the case. How should I proceed? Any hints will be appreciated!
approximation's user avatar
2 votes
3 answers
534 views

Limit in distributions of $\frac{\sin(tx)}{x}$

How can I find the limit of $\frac{\sin(tx)}{x}$ as $t \to \infty$ in $D'$ ? I understand that i need to see the $\lim_{t \to \infty}{\int_{\infty}^{\infty}{\frac{\sin(tx)\phi(x)}{x}dx}}$ for every ...
matvey kormushkin's user avatar
2 votes
1 answer
612 views

What is the asymptotic form of the spherical Bessel function $j_{n}(x)$ when $n \to \infty$?

I am trying to find the asymptotics of the spherical Bessel function $j_n(x)$ when $n\to \infty$. I was able to find something like $$ j_{n} ( x) \sim \sqrt{\frac{\pi}{2n}} \delta \left(x - n\...
z.v.'s user avatar
  • 652
2 votes
1 answer
272 views

Criteria for a sequence of funcions converge to Dirac Delta

I have been stuck with the following problem: Let $(f_n)_{n\in\mathbb{N}}$ be a sequence of integrable functions in $\mathbb{R}^N$ that satisfies: $$\int_{\mathbb{R}^N}f_n(x)dx = 1 \quad \forall n \...
Rudá Lima da Floresta's user avatar
2 votes
1 answer
143 views

Gelfand triples/Rigged Hilbert space notation

Usually the rigged Hilbert space is denoted by $\mathcal{S} \subset L^2 \subset \mathcal{S}'$, where $L^2$ is a Hilbert space (square integrable functions), $\mathcal{S}$ is the Schwartz space and $\...
Ali's user avatar
  • 357
2 votes
1 answer
58 views

Test functions and distributions

Let $a \in C^\infty(\Bbb R)$. Show that if $g \in D' (\Bbb R)$ satisfies $g' − ag = 0$, then $g \in C^\infty(\Bbb R)$, i.e. $g$ is a regular distribution associated with $C^\infty$ function. I have ...
MicroscooterMan's user avatar
2 votes
1 answer
82 views

Why do we have $\int_{t_0}^t \delta(t-t') c(t')=\frac{1}{2} c(t)$?

I am reading a physics paper in which they claim that: $$\int_{t_0}^t \delta(t-t') c(t')=\frac{1}{2} c(t).$$ They just say about this equation: "The result will always hold when $\int_{-\infty}^...
StarBucK's user avatar
  • 689
2 votes
1 answer
1k views

Find weak form of linear transport equation

I am stuck on the following problem that says: a) Find a weak formulation for the partial differential equation $${\partial u\over\partial t\ }+ c{\partial u \over \partial x\ }=0$$ b) Show ...
Galymbek's user avatar
  • 163
2 votes
3 answers
3k views

If X is a non-negative continuous random variable, show that $E[X]=\int_0^\infty (1-F(x)) dx$

There is a hint to solve this by using integration by parts. So I have $$ u = x \space \mbox{then} \space u'=1$$ $$v' = F(x) -1 \space \mbox{then} \space = ? $$ QUESTION: What is $v'$ ? What is ...
user1607's user avatar
  • 481
2 votes
2 answers
67 views

Why $\frac{\varepsilon}{x^2+\varepsilon^2}$ converges in the sense of distributions to a constant times the Dirac delta

The integral of $f_\varepsilon(x)=\frac\varepsilon{x^2+\varepsilon^2}$ is the tan inverse, which is well behaved anywhere on $\mathbb{R}$, and so $f_\varepsilon$ is in $L^1_\text{loc}(\mathbb{R})$. ...
manifoldcurious's user avatar
2 votes
1 answer
524 views

Extension of a distribution

Let $\Omega \subseteq \mathbb R^n$ be an open convex set and $T \in D'(\Omega)$. Is it possible to extend $T$ to a distribution in $D'(\mathbb R^n)$? I've been looking in some texts (Rudin and this ...
Gonzalo Benavides's user avatar
2 votes
1 answer
626 views

CDF for exponential family

Might be a stupid question though... we know for Exponential Family, we have the density is $$f(x|\theta)=h(x)\exp[\eta(\theta)\cdot T(x)-A(\theta)]$$ Is there a general form for the CDF function? I ...
Matata's user avatar
  • 2,088
2 votes
1 answer
316 views

In what sense are distributions more singular than functions?

Chapter 9 in Folland's Real Analysis starts with the following sentence: "At least as far back as Heaviside in the $1890$s, engineers and physicists have found it convenient to consider ...
Jonatan B. Bastos's user avatar
2 votes
1 answer
76 views

Proving existence of a Brownian functional

Let $W$ be a standard Brownian Motion indexed by $\mathbb{R}$, i.e., $W\restriction_{\mathopen{[}0,\infty\mathclose{[}}$ is a standard Brownian Motion, and $W\restriction_{\mathopen{]}-\infty,0\...
sami.spricht.sprache's user avatar
2 votes
2 answers
6k views

Formal derivation of the Fourier transform of Dirac delta using a distribution

The Fourier transform of Dirac delta is often naively calculated by considering Delta function as a function that makes sense within an integral and by using its fundamental property: $$ \int_{-\...
John's user avatar
  • 405
2 votes
2 answers
180 views

Is there a sense in which this limit is zero?

This question arose from this answer of mine on DSP.SE. Basically, I need to check if this is true: $$\lim_{M\to \infty} j\frac{\cos[\omega(M+1/2)]}{2\sin(\omega/2)}\stackrel{?}{=}0$$ where $j$ is ...
Tendero's user avatar
  • 798
2 votes
2 answers
476 views

Prove that , $e^{f(x)}-1$ is a schwartz class function when $f$ is a Schwartz function.

Suppose $f$ is a Schwartz function, prove that $e^{f(x)}-1$ is also a Schwartz function. In order to prove the result, we have to show that $|(e^{f(x)}-1)^{(n)}|\leq \frac{C_{m,n}}{|x|^{m}}$ for any ...
Jack's user avatar
  • 2,017
2 votes
1 answer
277 views

Laplace(-Beltrami) operator in the sense of distributions

Let us set ourselves in $\mathbb R^d$ and let $\Omega$ be an open set. Let $P$ be a differential operator with coefficients that admits a fundamental solution $E$ which is $C^\infty$ outside of $0$. ...
Marko Karbevski's user avatar
2 votes
1 answer
414 views

Multiplication of two distributions whose singular supports are disjoint

The Wikipedia page on distributions says that multiplication of two distributions whose singular supports are disjoint is easy to define, but the page doesn't actually define it. I'm not sure how to ...
leeto's user avatar
  • 681
2 votes
1 answer
94 views

Showing that $f(z)$ is analytic.

Let $x\in \Bbb R^n, y\in\Bbb R$, and $z\in\Bbb C$. For a fixed $\lambda\in\Bbb R^n$ and a smooth, compactly supported function $\varphi\in\mathcal D(\Bbb R^{n+1})$, I want to show that $$ f(z):=\int_{\...
BigbearZzz's user avatar
  • 15.1k
2 votes
1 answer
123 views

Fourier Transform of $\sin(\ln(t))u(t) $

Can someone help me with the example of Fourier transform $$\mathcal{F}[\sin(ln(t))u(t)](\omega)\ = \ \ ?$$ I know that there is a formula in the Table of Fourier Transform Pairs but it is only for $...
panzer055's user avatar
2 votes
1 answer
188 views

General theory of divergent, highly oscillatory integrals

I'm interested in evaluation of integrals of the form $$\int e^{i S(x)}O(x) dx,$$ where $S(x)$ is a polynomial of degree higher than one and $O(x)$ is a polynomial or more generally, polynomialy ...
Blazej's user avatar
  • 3,070

1
19 20
21
22 23
73